Howdy, Stranger!

It looks like you're new here. If you want to get involved, click one of these buttons!

What's the most LSAT way to weaken/strengthen this commonly featured argument?

Regis_Phalange63Regis_Phalange63 Alum Member
in General 1058 karma

premise: Group A took medicine X.
premise: Group B did not take medicine X.
BUT identical results were found in both groups.
conclusion: Hence, X is ineffective.

I'm trying to approach this in a formulaic way but I always find myself being led by answer choices. What do you think is the most LSAT way to strengthen(assumption)/weaken the argument above?

Comments

  • LouislepauvreLouislepauvre Alum Member
    750 karma

    In 7Sage’s webinars (which don’t get the hype they deserve) there is one called “Weaken/Strengthen” by Nicole Hopkins. She breaks these question types down pretty much mathematically. It transformed my approach to these questions. You gotta see it.

    I’ll paraphrase the webinar though. To me, this is a reverse causation argument. A does not cause B (Medicine X does not cause results). To strengthen that, you’d want to give more evidence of non-causation (kinda boring). To weaken it (much more fun), you’d introduce an alternative variable, C, which caused the non-causation.

    Group A has a more severe case of the disease.
    Group A was born with this disease, whereas Group B recently developed it.
    Group A also had the flu, whereas Group B didn’t.

    The right answer in this situation would most likely introduce alternative causes which would call into question the hypothesis that the medicine is ineffective.

    Seriously check out Nicole’s webinar! She goes over Correlation/Causation arguments, argument by analogy, and phenomenon/hypothesis argument. Very thoughtful and helpful. Good luck!

  • Cant Get RightCant Get Right Yearly + Live Member Sage 🍌 7Sage Tutor
    27821 karma

    Yeah, basically anything that says we shouldn’t expect identical outcomes:

    Group A smokes an average two packs a day; Group B are all vegan fitness instructors.

  • Regis_Phalange63Regis_Phalange63 Alum Member
    1058 karma

    @Louislepauvre @"Cant Get Right" Thanks so much! I'll definitely check out Nicole's webinar. :)

  • redshiftredshift Alum Member
    edited November 2018 261 karma

    The big assumption here is that the author is assuming that finding identical results in both groups is sufficient to determine that the medicine was ineffective. However, as Can’t Get Right said, the composition of the two groups could be flawed in some way. If the medicine hypothetically brings people to 100% lung capacity, it would obviously have more effect on a group of smokers operating at 50% lung capacity vs on a group of marathons runners already operation at 100.

    As we know from the 7Sage CC, to weaken we deny the assumption, to strengthen we affirm it.

    So a weaken answer here might be: “The study drawed from two groups of people - one, smokers, the other, non-smokers.” Note that this does not destroy the argument, but it weakens it.

    A strengthen answer here might be: “The study drawed from randomly selected groups and was carefully conducted under the supervision of experts.”

Sign In or Register to comment.